Showing that $x^2+5=y^3$ has no integer solutions.












2












$begingroup$


I'm trying to show that the Diophantine equation $x^2+5=y^3$ has no integer solutions using the fact that $mathbb Z[ sqrt{-5}]$ has class number two. I think I have the general idea, but I'm having a tough time establishing the statement in bold below. Any ideas?



The equation gives a factorization into ideals
$$
(x+sqrt{-5})(x-sqrt{-5})=(y)^3
$$

in $mathbb Z[ sqrt{-5}]$. Assuming that $(x+sqrt{-5})$ and $(x-sqrt{-5})$ are relatively prime, we know that $(x+sqrt{-5})=mathfrak a^3$ and $(x+sqrt{-5})=mathfrak b^3$ for some ideals $mathfrak a, mathfrak b$ in $mathbb Z[ sqrt{-5}]$. Then the classes of $mathfrak a$ and $mathfrak b$ have order dividing 3 in the class group, hence they must both be principal, generated by some $alpha,betain mathbb Z[sqrt{-5}]$, respectively, as the class number is 2. Hence $alpha^3=x+sqrt{-5}$ up to units ($pm 1$), and matching up real and imaginary parts then gives a contradiction.



How can I show that the ideals $(x+sqrt{-5})$ and $(x-sqrt{-5})$ are relatively prime? I was trying something along the lines of letting $mathfrak p$ be a prime of $mathbb Z[sqrt{-5}]$ diving both ideals, in which case it also divides their sum (gcd). Then $$2sqrt{-5}=x+sqrt{-5}-(x-sqrt{-5})in (x+sqrt{-5},x-sqrt{-5})subseteq mathfrak p,$$ so $mathfrak pmid (2sqrt{-5})$... But I'm not totally sure where I'm going with this.










share|cite|improve this question









$endgroup$








  • 3




    $begingroup$
    Just realized this could be a duplicate of: math.stackexchange.com/questions/1239090/…. Will read this solution to see if it answers my question...
    $endgroup$
    – Arbutus
    Jan 25 at 16:05








  • 1




    $begingroup$
    The ring $mathbb{Z}[sqrt{-5}]$ is not a UFD.
    $endgroup$
    – egreg
    Jan 25 at 16:12






  • 3




    $begingroup$
    Right, but it is a Dedekind domain, so we have unique factorization of ideals.
    $endgroup$
    – Arbutus
    Jan 25 at 16:12










  • $begingroup$
    That's indeed the case.
    $endgroup$
    – egreg
    Jan 25 at 16:13










  • $begingroup$
    So, I don't think my question is a duplicate: in the link above, @KCd's comment essentially explains the method outlined above, but does not shed light on how to show the ideals are relatively prime.
    $endgroup$
    – Arbutus
    Jan 25 at 16:17
















2












$begingroup$


I'm trying to show that the Diophantine equation $x^2+5=y^3$ has no integer solutions using the fact that $mathbb Z[ sqrt{-5}]$ has class number two. I think I have the general idea, but I'm having a tough time establishing the statement in bold below. Any ideas?



The equation gives a factorization into ideals
$$
(x+sqrt{-5})(x-sqrt{-5})=(y)^3
$$

in $mathbb Z[ sqrt{-5}]$. Assuming that $(x+sqrt{-5})$ and $(x-sqrt{-5})$ are relatively prime, we know that $(x+sqrt{-5})=mathfrak a^3$ and $(x+sqrt{-5})=mathfrak b^3$ for some ideals $mathfrak a, mathfrak b$ in $mathbb Z[ sqrt{-5}]$. Then the classes of $mathfrak a$ and $mathfrak b$ have order dividing 3 in the class group, hence they must both be principal, generated by some $alpha,betain mathbb Z[sqrt{-5}]$, respectively, as the class number is 2. Hence $alpha^3=x+sqrt{-5}$ up to units ($pm 1$), and matching up real and imaginary parts then gives a contradiction.



How can I show that the ideals $(x+sqrt{-5})$ and $(x-sqrt{-5})$ are relatively prime? I was trying something along the lines of letting $mathfrak p$ be a prime of $mathbb Z[sqrt{-5}]$ diving both ideals, in which case it also divides their sum (gcd). Then $$2sqrt{-5}=x+sqrt{-5}-(x-sqrt{-5})in (x+sqrt{-5},x-sqrt{-5})subseteq mathfrak p,$$ so $mathfrak pmid (2sqrt{-5})$... But I'm not totally sure where I'm going with this.










share|cite|improve this question









$endgroup$








  • 3




    $begingroup$
    Just realized this could be a duplicate of: math.stackexchange.com/questions/1239090/…. Will read this solution to see if it answers my question...
    $endgroup$
    – Arbutus
    Jan 25 at 16:05








  • 1




    $begingroup$
    The ring $mathbb{Z}[sqrt{-5}]$ is not a UFD.
    $endgroup$
    – egreg
    Jan 25 at 16:12






  • 3




    $begingroup$
    Right, but it is a Dedekind domain, so we have unique factorization of ideals.
    $endgroup$
    – Arbutus
    Jan 25 at 16:12










  • $begingroup$
    That's indeed the case.
    $endgroup$
    – egreg
    Jan 25 at 16:13










  • $begingroup$
    So, I don't think my question is a duplicate: in the link above, @KCd's comment essentially explains the method outlined above, but does not shed light on how to show the ideals are relatively prime.
    $endgroup$
    – Arbutus
    Jan 25 at 16:17














2












2








2


3



$begingroup$


I'm trying to show that the Diophantine equation $x^2+5=y^3$ has no integer solutions using the fact that $mathbb Z[ sqrt{-5}]$ has class number two. I think I have the general idea, but I'm having a tough time establishing the statement in bold below. Any ideas?



The equation gives a factorization into ideals
$$
(x+sqrt{-5})(x-sqrt{-5})=(y)^3
$$

in $mathbb Z[ sqrt{-5}]$. Assuming that $(x+sqrt{-5})$ and $(x-sqrt{-5})$ are relatively prime, we know that $(x+sqrt{-5})=mathfrak a^3$ and $(x+sqrt{-5})=mathfrak b^3$ for some ideals $mathfrak a, mathfrak b$ in $mathbb Z[ sqrt{-5}]$. Then the classes of $mathfrak a$ and $mathfrak b$ have order dividing 3 in the class group, hence they must both be principal, generated by some $alpha,betain mathbb Z[sqrt{-5}]$, respectively, as the class number is 2. Hence $alpha^3=x+sqrt{-5}$ up to units ($pm 1$), and matching up real and imaginary parts then gives a contradiction.



How can I show that the ideals $(x+sqrt{-5})$ and $(x-sqrt{-5})$ are relatively prime? I was trying something along the lines of letting $mathfrak p$ be a prime of $mathbb Z[sqrt{-5}]$ diving both ideals, in which case it also divides their sum (gcd). Then $$2sqrt{-5}=x+sqrt{-5}-(x-sqrt{-5})in (x+sqrt{-5},x-sqrt{-5})subseteq mathfrak p,$$ so $mathfrak pmid (2sqrt{-5})$... But I'm not totally sure where I'm going with this.










share|cite|improve this question









$endgroup$




I'm trying to show that the Diophantine equation $x^2+5=y^3$ has no integer solutions using the fact that $mathbb Z[ sqrt{-5}]$ has class number two. I think I have the general idea, but I'm having a tough time establishing the statement in bold below. Any ideas?



The equation gives a factorization into ideals
$$
(x+sqrt{-5})(x-sqrt{-5})=(y)^3
$$

in $mathbb Z[ sqrt{-5}]$. Assuming that $(x+sqrt{-5})$ and $(x-sqrt{-5})$ are relatively prime, we know that $(x+sqrt{-5})=mathfrak a^3$ and $(x+sqrt{-5})=mathfrak b^3$ for some ideals $mathfrak a, mathfrak b$ in $mathbb Z[ sqrt{-5}]$. Then the classes of $mathfrak a$ and $mathfrak b$ have order dividing 3 in the class group, hence they must both be principal, generated by some $alpha,betain mathbb Z[sqrt{-5}]$, respectively, as the class number is 2. Hence $alpha^3=x+sqrt{-5}$ up to units ($pm 1$), and matching up real and imaginary parts then gives a contradiction.



How can I show that the ideals $(x+sqrt{-5})$ and $(x-sqrt{-5})$ are relatively prime? I was trying something along the lines of letting $mathfrak p$ be a prime of $mathbb Z[sqrt{-5}]$ diving both ideals, in which case it also divides their sum (gcd). Then $$2sqrt{-5}=x+sqrt{-5}-(x-sqrt{-5})in (x+sqrt{-5},x-sqrt{-5})subseteq mathfrak p,$$ so $mathfrak pmid (2sqrt{-5})$... But I'm not totally sure where I'm going with this.







number-theory algebraic-number-theory diophantine-equations dedekind-domain






share|cite|improve this question













share|cite|improve this question











share|cite|improve this question




share|cite|improve this question










asked Jan 25 at 16:03









ArbutusArbutus

723715




723715








  • 3




    $begingroup$
    Just realized this could be a duplicate of: math.stackexchange.com/questions/1239090/…. Will read this solution to see if it answers my question...
    $endgroup$
    – Arbutus
    Jan 25 at 16:05








  • 1




    $begingroup$
    The ring $mathbb{Z}[sqrt{-5}]$ is not a UFD.
    $endgroup$
    – egreg
    Jan 25 at 16:12






  • 3




    $begingroup$
    Right, but it is a Dedekind domain, so we have unique factorization of ideals.
    $endgroup$
    – Arbutus
    Jan 25 at 16:12










  • $begingroup$
    That's indeed the case.
    $endgroup$
    – egreg
    Jan 25 at 16:13










  • $begingroup$
    So, I don't think my question is a duplicate: in the link above, @KCd's comment essentially explains the method outlined above, but does not shed light on how to show the ideals are relatively prime.
    $endgroup$
    – Arbutus
    Jan 25 at 16:17














  • 3




    $begingroup$
    Just realized this could be a duplicate of: math.stackexchange.com/questions/1239090/…. Will read this solution to see if it answers my question...
    $endgroup$
    – Arbutus
    Jan 25 at 16:05








  • 1




    $begingroup$
    The ring $mathbb{Z}[sqrt{-5}]$ is not a UFD.
    $endgroup$
    – egreg
    Jan 25 at 16:12






  • 3




    $begingroup$
    Right, but it is a Dedekind domain, so we have unique factorization of ideals.
    $endgroup$
    – Arbutus
    Jan 25 at 16:12










  • $begingroup$
    That's indeed the case.
    $endgroup$
    – egreg
    Jan 25 at 16:13










  • $begingroup$
    So, I don't think my question is a duplicate: in the link above, @KCd's comment essentially explains the method outlined above, but does not shed light on how to show the ideals are relatively prime.
    $endgroup$
    – Arbutus
    Jan 25 at 16:17








3




3




$begingroup$
Just realized this could be a duplicate of: math.stackexchange.com/questions/1239090/…. Will read this solution to see if it answers my question...
$endgroup$
– Arbutus
Jan 25 at 16:05






$begingroup$
Just realized this could be a duplicate of: math.stackexchange.com/questions/1239090/…. Will read this solution to see if it answers my question...
$endgroup$
– Arbutus
Jan 25 at 16:05






1




1




$begingroup$
The ring $mathbb{Z}[sqrt{-5}]$ is not a UFD.
$endgroup$
– egreg
Jan 25 at 16:12




$begingroup$
The ring $mathbb{Z}[sqrt{-5}]$ is not a UFD.
$endgroup$
– egreg
Jan 25 at 16:12




3




3




$begingroup$
Right, but it is a Dedekind domain, so we have unique factorization of ideals.
$endgroup$
– Arbutus
Jan 25 at 16:12




$begingroup$
Right, but it is a Dedekind domain, so we have unique factorization of ideals.
$endgroup$
– Arbutus
Jan 25 at 16:12












$begingroup$
That's indeed the case.
$endgroup$
– egreg
Jan 25 at 16:13




$begingroup$
That's indeed the case.
$endgroup$
– egreg
Jan 25 at 16:13












$begingroup$
So, I don't think my question is a duplicate: in the link above, @KCd's comment essentially explains the method outlined above, but does not shed light on how to show the ideals are relatively prime.
$endgroup$
– Arbutus
Jan 25 at 16:17




$begingroup$
So, I don't think my question is a duplicate: in the link above, @KCd's comment essentially explains the method outlined above, but does not shed light on how to show the ideals are relatively prime.
$endgroup$
– Arbutus
Jan 25 at 16:17










4 Answers
4






active

oldest

votes


















2












$begingroup$

$mathfrak pmid(2sqrt{-5})$, so $mathfrak pmid(2)$ or $mathfrak pmid(sqrt{-5})$. Since $(2),(sqrt{-5})$ are prime, $mathfrak p$ is equal to one of them. We can't have $mathfrak p=(2)$, since $2nmid x+sqrt{-5}$, so $mathfrak p=(sqrt{-5})$, so $sqrt{-5}mid x+sqrt{-5}$. This implies $5mid x$, and you can derive a contradiction from $x^2+5=y^3$.



EDIT: As Yong Hao Ng notes in the comment, $(2)$ is not a prime ideal - it factors as $(2,1+sqrt{-5})^2$, so we get that $mathfrak p=(2,1+sqrt{-5})$. Then $x+sqrt{-5}in(2,1+sqrt{-5})$, so $x-1in(2,1+sqrt{-5})$ and we get $x$ is odd. Now we get a contradiction from $x^2+5equiv 2pmod 4$, since $y^3$ can't be even but indivisible by $4$.






share|cite|improve this answer











$endgroup$













  • $begingroup$
    @Servaes That isn't right - this is true in any ring as long as $mathfrak p$ is prime, at least assuming we define $Imid J$ as $Jsubseteq I$. We don't need $(2),(sqrt{-5})$ prime for this either.
    $endgroup$
    – Wojowu
    Jan 27 at 13:34










  • $begingroup$
    @Wojuwu You are right, sorry for the confusion.
    $endgroup$
    – Servaes
    Jan 27 at 13:43










  • $begingroup$
    (+1) but I'm wondering if $(2)$ isn't prime and $$(2)=(2,1-sqrt 5)^2$$ instead. The ring mod $(2)$, $mathbb Z[sqrt 5]/(2)$, has the residue classes ${0,1,sqrt 5,1+sqrt 5}$ and there is a zero divisor since $$(1+sqrt 5)^2 = 6+2sqrt 5 equiv 0pmod 2$$ But the residue ring should be an integral domain.
    $endgroup$
    – Yong Hao Ng
    Jan 27 at 17:33










  • $begingroup$
    @YongHaoNg I'm afraid you are right (except we are in $mathbb Z[sqrt{-5}]$, not $mathbb Z[sqrt{5}]$). I will edit accordingly.
    $endgroup$
    – Wojowu
    Jan 27 at 18:48










  • $begingroup$
    @Wojowu I'm afraid I don't understand your argument on the odness of $x$. As I interpret it, you write $x-1$ under the form $2a+b(1-sqrt{-5})$, but here $a, b in mathbf Z[sqrt {-5}]$, so I think you can't conclude a priori. Did I miss something ?
    $endgroup$
    – nguyen quang do
    Jan 27 at 22:09



















3












$begingroup$

This has been answered many times on this site, so it is certainly a duplicate. On the other hand, it may be still worth to give the link to a very nice lecture note by Keith Conrad here, where he does proofs for many interesting cases of the Mordell equation $y^2=x^3-n$. For $n=5$ there are no integer solutions. More references here are found at OEIS A054504.






share|cite|improve this answer









$endgroup$













  • $begingroup$
    In this post such ideals are considered in Arturo's answer, but there are still better posts, I suppose.
    $endgroup$
    – Dietrich Burde
    Jan 25 at 16:37












  • $begingroup$
    You were almost there.
    $endgroup$
    – nguyen quang do
    Jan 27 at 10:28



















1












$begingroup$

In the ideal factorization $(x+sqrt {-5})(x-sqrt {-5})=(y)^3$, "how can I show that the ideals $(x pm sqrt {-5})$ are relatively prime?" Suppose that $P$ is a common prime divisor, then $P^{2a}$ will appear in the LHS of the factorization. But $P^{3b}$ will appear in the RHS because of the cube, hence at least $P^6$ will divide both sides because of uniqueness. Following your argument, $P^3$ must divide $(2)(sqrt {-5})$. In the quadratic field $mathbf Q(sqrt {-5})$, we know that the prime $5$ is totally ramified, and $(5)=(sqrt {-5})^2$ shows that $(sqrt {-5})$ is prime. Besides $(2)=Q^2$, with $Q=(2, 1+sqrt {-5})$ (so the class of $Q$ is the unique class of order $2$ of the class group). It follows again by uniqueness of factorization that $P^3$ cannot divide $(sqrt {-5})Q^2$, and we are done.






share|cite|improve this answer









$endgroup$





















    1












    $begingroup$

    If ideals $I,J$ are relatively prime then their sum generates the ring $R$, so we can show this. In particular we can show also that
    $$
    (a + bsqrt {- 5})(x+ sqrt{- 5}) + (c + dsqrt {- 5})(x - sqrt {- 5}) = 1
    $$

    for some $a,b,c,d$.



    This gives the equations
    $$
    begin{align}
    (a+c)x - 5(b-d) &= 1\
    (a-c) + (b+d)x &= 0
    end{align}
    $$

    So a necessary condition is $5$ does not divide $x$. This can be checked to be true from $x^2 + 5 = y^3$, but otherwise the ideals are not necessarily relatively prime in general.





    Setting
    $$
    c = a + (b+d)x
    $$

    gives us
    $$
    2ax = (1+5(b-d)) - (b+d)x^2
    $$

    Taking $pmod 2$ we see that $x$ must be even, so to get an integer solution for $a$ we must have $1+5(b-d)$ divisible by $2x$.



    The obvious way is to look at the linear diophantine solutions
    $$
    xu - 5v = 1
    $$

    We can always choose $u$ to be even, so now we can set $d=0, v=b$ to get
    $$
    begin{align}
    a &= frac{1+5(b-d)}{2x} - (b+d)frac{x}{2}\
    &= frac{1+5v}{2x} - vfrac{x}{2}\
    &= frac{u}{2} - vfrac{x}{2}
    end{align}
    $$

    Since $u/2$ is integral we just need $vx$ to be even, which is true based on an earlier assumption: $x$ must be even.





    It remains to show that $x$ is indeed even.
    If $x=2m+1$ is odd, then $y$ is even and
    $$
    x^2 + 5 = 4m^2+4m+ 6 equiv 2pmod 4
    $$

    which contradicts $y^3 equiv 0 pmod 4$. Hence $x$ is even and we are done, finding a solution to the initial equation as
    $$
    (u/2 -vx/2 + vsqrt {-5})(x + sqrt {-5}) + (u/2 + vx/2)(x - sqrt {-5}) = 1
    $$

    (Recall that $xu-5v=1$ and $u$ chosen to be even.)



    Since the two ideals generate $1$, they are relatively prime.






    share|cite|improve this answer











    $endgroup$













      Your Answer





      StackExchange.ifUsing("editor", function () {
      return StackExchange.using("mathjaxEditing", function () {
      StackExchange.MarkdownEditor.creationCallbacks.add(function (editor, postfix) {
      StackExchange.mathjaxEditing.prepareWmdForMathJax(editor, postfix, [["$", "$"], ["\\(","\\)"]]);
      });
      });
      }, "mathjax-editing");

      StackExchange.ready(function() {
      var channelOptions = {
      tags: "".split(" "),
      id: "69"
      };
      initTagRenderer("".split(" "), "".split(" "), channelOptions);

      StackExchange.using("externalEditor", function() {
      // Have to fire editor after snippets, if snippets enabled
      if (StackExchange.settings.snippets.snippetsEnabled) {
      StackExchange.using("snippets", function() {
      createEditor();
      });
      }
      else {
      createEditor();
      }
      });

      function createEditor() {
      StackExchange.prepareEditor({
      heartbeatType: 'answer',
      autoActivateHeartbeat: false,
      convertImagesToLinks: true,
      noModals: true,
      showLowRepImageUploadWarning: true,
      reputationToPostImages: 10,
      bindNavPrevention: true,
      postfix: "",
      imageUploader: {
      brandingHtml: "Powered by u003ca class="icon-imgur-white" href="https://imgur.com/"u003eu003c/au003e",
      contentPolicyHtml: "User contributions licensed under u003ca href="https://creativecommons.org/licenses/by-sa/3.0/"u003ecc by-sa 3.0 with attribution requiredu003c/au003e u003ca href="https://stackoverflow.com/legal/content-policy"u003e(content policy)u003c/au003e",
      allowUrls: true
      },
      noCode: true, onDemand: true,
      discardSelector: ".discard-answer"
      ,immediatelyShowMarkdownHelp:true
      });


      }
      });














      draft saved

      draft discarded


















      StackExchange.ready(
      function () {
      StackExchange.openid.initPostLogin('.new-post-login', 'https%3a%2f%2fmath.stackexchange.com%2fquestions%2f3087254%2fshowing-that-x25-y3-has-no-integer-solutions%23new-answer', 'question_page');
      }
      );

      Post as a guest















      Required, but never shown

























      4 Answers
      4






      active

      oldest

      votes








      4 Answers
      4






      active

      oldest

      votes









      active

      oldest

      votes






      active

      oldest

      votes









      2












      $begingroup$

      $mathfrak pmid(2sqrt{-5})$, so $mathfrak pmid(2)$ or $mathfrak pmid(sqrt{-5})$. Since $(2),(sqrt{-5})$ are prime, $mathfrak p$ is equal to one of them. We can't have $mathfrak p=(2)$, since $2nmid x+sqrt{-5}$, so $mathfrak p=(sqrt{-5})$, so $sqrt{-5}mid x+sqrt{-5}$. This implies $5mid x$, and you can derive a contradiction from $x^2+5=y^3$.



      EDIT: As Yong Hao Ng notes in the comment, $(2)$ is not a prime ideal - it factors as $(2,1+sqrt{-5})^2$, so we get that $mathfrak p=(2,1+sqrt{-5})$. Then $x+sqrt{-5}in(2,1+sqrt{-5})$, so $x-1in(2,1+sqrt{-5})$ and we get $x$ is odd. Now we get a contradiction from $x^2+5equiv 2pmod 4$, since $y^3$ can't be even but indivisible by $4$.






      share|cite|improve this answer











      $endgroup$













      • $begingroup$
        @Servaes That isn't right - this is true in any ring as long as $mathfrak p$ is prime, at least assuming we define $Imid J$ as $Jsubseteq I$. We don't need $(2),(sqrt{-5})$ prime for this either.
        $endgroup$
        – Wojowu
        Jan 27 at 13:34










      • $begingroup$
        @Wojuwu You are right, sorry for the confusion.
        $endgroup$
        – Servaes
        Jan 27 at 13:43










      • $begingroup$
        (+1) but I'm wondering if $(2)$ isn't prime and $$(2)=(2,1-sqrt 5)^2$$ instead. The ring mod $(2)$, $mathbb Z[sqrt 5]/(2)$, has the residue classes ${0,1,sqrt 5,1+sqrt 5}$ and there is a zero divisor since $$(1+sqrt 5)^2 = 6+2sqrt 5 equiv 0pmod 2$$ But the residue ring should be an integral domain.
        $endgroup$
        – Yong Hao Ng
        Jan 27 at 17:33










      • $begingroup$
        @YongHaoNg I'm afraid you are right (except we are in $mathbb Z[sqrt{-5}]$, not $mathbb Z[sqrt{5}]$). I will edit accordingly.
        $endgroup$
        – Wojowu
        Jan 27 at 18:48










      • $begingroup$
        @Wojowu I'm afraid I don't understand your argument on the odness of $x$. As I interpret it, you write $x-1$ under the form $2a+b(1-sqrt{-5})$, but here $a, b in mathbf Z[sqrt {-5}]$, so I think you can't conclude a priori. Did I miss something ?
        $endgroup$
        – nguyen quang do
        Jan 27 at 22:09
















      2












      $begingroup$

      $mathfrak pmid(2sqrt{-5})$, so $mathfrak pmid(2)$ or $mathfrak pmid(sqrt{-5})$. Since $(2),(sqrt{-5})$ are prime, $mathfrak p$ is equal to one of them. We can't have $mathfrak p=(2)$, since $2nmid x+sqrt{-5}$, so $mathfrak p=(sqrt{-5})$, so $sqrt{-5}mid x+sqrt{-5}$. This implies $5mid x$, and you can derive a contradiction from $x^2+5=y^3$.



      EDIT: As Yong Hao Ng notes in the comment, $(2)$ is not a prime ideal - it factors as $(2,1+sqrt{-5})^2$, so we get that $mathfrak p=(2,1+sqrt{-5})$. Then $x+sqrt{-5}in(2,1+sqrt{-5})$, so $x-1in(2,1+sqrt{-5})$ and we get $x$ is odd. Now we get a contradiction from $x^2+5equiv 2pmod 4$, since $y^3$ can't be even but indivisible by $4$.






      share|cite|improve this answer











      $endgroup$













      • $begingroup$
        @Servaes That isn't right - this is true in any ring as long as $mathfrak p$ is prime, at least assuming we define $Imid J$ as $Jsubseteq I$. We don't need $(2),(sqrt{-5})$ prime for this either.
        $endgroup$
        – Wojowu
        Jan 27 at 13:34










      • $begingroup$
        @Wojuwu You are right, sorry for the confusion.
        $endgroup$
        – Servaes
        Jan 27 at 13:43










      • $begingroup$
        (+1) but I'm wondering if $(2)$ isn't prime and $$(2)=(2,1-sqrt 5)^2$$ instead. The ring mod $(2)$, $mathbb Z[sqrt 5]/(2)$, has the residue classes ${0,1,sqrt 5,1+sqrt 5}$ and there is a zero divisor since $$(1+sqrt 5)^2 = 6+2sqrt 5 equiv 0pmod 2$$ But the residue ring should be an integral domain.
        $endgroup$
        – Yong Hao Ng
        Jan 27 at 17:33










      • $begingroup$
        @YongHaoNg I'm afraid you are right (except we are in $mathbb Z[sqrt{-5}]$, not $mathbb Z[sqrt{5}]$). I will edit accordingly.
        $endgroup$
        – Wojowu
        Jan 27 at 18:48










      • $begingroup$
        @Wojowu I'm afraid I don't understand your argument on the odness of $x$. As I interpret it, you write $x-1$ under the form $2a+b(1-sqrt{-5})$, but here $a, b in mathbf Z[sqrt {-5}]$, so I think you can't conclude a priori. Did I miss something ?
        $endgroup$
        – nguyen quang do
        Jan 27 at 22:09














      2












      2








      2





      $begingroup$

      $mathfrak pmid(2sqrt{-5})$, so $mathfrak pmid(2)$ or $mathfrak pmid(sqrt{-5})$. Since $(2),(sqrt{-5})$ are prime, $mathfrak p$ is equal to one of them. We can't have $mathfrak p=(2)$, since $2nmid x+sqrt{-5}$, so $mathfrak p=(sqrt{-5})$, so $sqrt{-5}mid x+sqrt{-5}$. This implies $5mid x$, and you can derive a contradiction from $x^2+5=y^3$.



      EDIT: As Yong Hao Ng notes in the comment, $(2)$ is not a prime ideal - it factors as $(2,1+sqrt{-5})^2$, so we get that $mathfrak p=(2,1+sqrt{-5})$. Then $x+sqrt{-5}in(2,1+sqrt{-5})$, so $x-1in(2,1+sqrt{-5})$ and we get $x$ is odd. Now we get a contradiction from $x^2+5equiv 2pmod 4$, since $y^3$ can't be even but indivisible by $4$.






      share|cite|improve this answer











      $endgroup$



      $mathfrak pmid(2sqrt{-5})$, so $mathfrak pmid(2)$ or $mathfrak pmid(sqrt{-5})$. Since $(2),(sqrt{-5})$ are prime, $mathfrak p$ is equal to one of them. We can't have $mathfrak p=(2)$, since $2nmid x+sqrt{-5}$, so $mathfrak p=(sqrt{-5})$, so $sqrt{-5}mid x+sqrt{-5}$. This implies $5mid x$, and you can derive a contradiction from $x^2+5=y^3$.



      EDIT: As Yong Hao Ng notes in the comment, $(2)$ is not a prime ideal - it factors as $(2,1+sqrt{-5})^2$, so we get that $mathfrak p=(2,1+sqrt{-5})$. Then $x+sqrt{-5}in(2,1+sqrt{-5})$, so $x-1in(2,1+sqrt{-5})$ and we get $x$ is odd. Now we get a contradiction from $x^2+5equiv 2pmod 4$, since $y^3$ can't be even but indivisible by $4$.







      share|cite|improve this answer














      share|cite|improve this answer



      share|cite|improve this answer








      edited Jan 27 at 18:50

























      answered Jan 27 at 13:06









      WojowuWojowu

      18.9k23173




      18.9k23173












      • $begingroup$
        @Servaes That isn't right - this is true in any ring as long as $mathfrak p$ is prime, at least assuming we define $Imid J$ as $Jsubseteq I$. We don't need $(2),(sqrt{-5})$ prime for this either.
        $endgroup$
        – Wojowu
        Jan 27 at 13:34










      • $begingroup$
        @Wojuwu You are right, sorry for the confusion.
        $endgroup$
        – Servaes
        Jan 27 at 13:43










      • $begingroup$
        (+1) but I'm wondering if $(2)$ isn't prime and $$(2)=(2,1-sqrt 5)^2$$ instead. The ring mod $(2)$, $mathbb Z[sqrt 5]/(2)$, has the residue classes ${0,1,sqrt 5,1+sqrt 5}$ and there is a zero divisor since $$(1+sqrt 5)^2 = 6+2sqrt 5 equiv 0pmod 2$$ But the residue ring should be an integral domain.
        $endgroup$
        – Yong Hao Ng
        Jan 27 at 17:33










      • $begingroup$
        @YongHaoNg I'm afraid you are right (except we are in $mathbb Z[sqrt{-5}]$, not $mathbb Z[sqrt{5}]$). I will edit accordingly.
        $endgroup$
        – Wojowu
        Jan 27 at 18:48










      • $begingroup$
        @Wojowu I'm afraid I don't understand your argument on the odness of $x$. As I interpret it, you write $x-1$ under the form $2a+b(1-sqrt{-5})$, but here $a, b in mathbf Z[sqrt {-5}]$, so I think you can't conclude a priori. Did I miss something ?
        $endgroup$
        – nguyen quang do
        Jan 27 at 22:09


















      • $begingroup$
        @Servaes That isn't right - this is true in any ring as long as $mathfrak p$ is prime, at least assuming we define $Imid J$ as $Jsubseteq I$. We don't need $(2),(sqrt{-5})$ prime for this either.
        $endgroup$
        – Wojowu
        Jan 27 at 13:34










      • $begingroup$
        @Wojuwu You are right, sorry for the confusion.
        $endgroup$
        – Servaes
        Jan 27 at 13:43










      • $begingroup$
        (+1) but I'm wondering if $(2)$ isn't prime and $$(2)=(2,1-sqrt 5)^2$$ instead. The ring mod $(2)$, $mathbb Z[sqrt 5]/(2)$, has the residue classes ${0,1,sqrt 5,1+sqrt 5}$ and there is a zero divisor since $$(1+sqrt 5)^2 = 6+2sqrt 5 equiv 0pmod 2$$ But the residue ring should be an integral domain.
        $endgroup$
        – Yong Hao Ng
        Jan 27 at 17:33










      • $begingroup$
        @YongHaoNg I'm afraid you are right (except we are in $mathbb Z[sqrt{-5}]$, not $mathbb Z[sqrt{5}]$). I will edit accordingly.
        $endgroup$
        – Wojowu
        Jan 27 at 18:48










      • $begingroup$
        @Wojowu I'm afraid I don't understand your argument on the odness of $x$. As I interpret it, you write $x-1$ under the form $2a+b(1-sqrt{-5})$, but here $a, b in mathbf Z[sqrt {-5}]$, so I think you can't conclude a priori. Did I miss something ?
        $endgroup$
        – nguyen quang do
        Jan 27 at 22:09
















      $begingroup$
      @Servaes That isn't right - this is true in any ring as long as $mathfrak p$ is prime, at least assuming we define $Imid J$ as $Jsubseteq I$. We don't need $(2),(sqrt{-5})$ prime for this either.
      $endgroup$
      – Wojowu
      Jan 27 at 13:34




      $begingroup$
      @Servaes That isn't right - this is true in any ring as long as $mathfrak p$ is prime, at least assuming we define $Imid J$ as $Jsubseteq I$. We don't need $(2),(sqrt{-5})$ prime for this either.
      $endgroup$
      – Wojowu
      Jan 27 at 13:34












      $begingroup$
      @Wojuwu You are right, sorry for the confusion.
      $endgroup$
      – Servaes
      Jan 27 at 13:43




      $begingroup$
      @Wojuwu You are right, sorry for the confusion.
      $endgroup$
      – Servaes
      Jan 27 at 13:43












      $begingroup$
      (+1) but I'm wondering if $(2)$ isn't prime and $$(2)=(2,1-sqrt 5)^2$$ instead. The ring mod $(2)$, $mathbb Z[sqrt 5]/(2)$, has the residue classes ${0,1,sqrt 5,1+sqrt 5}$ and there is a zero divisor since $$(1+sqrt 5)^2 = 6+2sqrt 5 equiv 0pmod 2$$ But the residue ring should be an integral domain.
      $endgroup$
      – Yong Hao Ng
      Jan 27 at 17:33




      $begingroup$
      (+1) but I'm wondering if $(2)$ isn't prime and $$(2)=(2,1-sqrt 5)^2$$ instead. The ring mod $(2)$, $mathbb Z[sqrt 5]/(2)$, has the residue classes ${0,1,sqrt 5,1+sqrt 5}$ and there is a zero divisor since $$(1+sqrt 5)^2 = 6+2sqrt 5 equiv 0pmod 2$$ But the residue ring should be an integral domain.
      $endgroup$
      – Yong Hao Ng
      Jan 27 at 17:33












      $begingroup$
      @YongHaoNg I'm afraid you are right (except we are in $mathbb Z[sqrt{-5}]$, not $mathbb Z[sqrt{5}]$). I will edit accordingly.
      $endgroup$
      – Wojowu
      Jan 27 at 18:48




      $begingroup$
      @YongHaoNg I'm afraid you are right (except we are in $mathbb Z[sqrt{-5}]$, not $mathbb Z[sqrt{5}]$). I will edit accordingly.
      $endgroup$
      – Wojowu
      Jan 27 at 18:48












      $begingroup$
      @Wojowu I'm afraid I don't understand your argument on the odness of $x$. As I interpret it, you write $x-1$ under the form $2a+b(1-sqrt{-5})$, but here $a, b in mathbf Z[sqrt {-5}]$, so I think you can't conclude a priori. Did I miss something ?
      $endgroup$
      – nguyen quang do
      Jan 27 at 22:09




      $begingroup$
      @Wojowu I'm afraid I don't understand your argument on the odness of $x$. As I interpret it, you write $x-1$ under the form $2a+b(1-sqrt{-5})$, but here $a, b in mathbf Z[sqrt {-5}]$, so I think you can't conclude a priori. Did I miss something ?
      $endgroup$
      – nguyen quang do
      Jan 27 at 22:09











      3












      $begingroup$

      This has been answered many times on this site, so it is certainly a duplicate. On the other hand, it may be still worth to give the link to a very nice lecture note by Keith Conrad here, where he does proofs for many interesting cases of the Mordell equation $y^2=x^3-n$. For $n=5$ there are no integer solutions. More references here are found at OEIS A054504.






      share|cite|improve this answer









      $endgroup$













      • $begingroup$
        In this post such ideals are considered in Arturo's answer, but there are still better posts, I suppose.
        $endgroup$
        – Dietrich Burde
        Jan 25 at 16:37












      • $begingroup$
        You were almost there.
        $endgroup$
        – nguyen quang do
        Jan 27 at 10:28
















      3












      $begingroup$

      This has been answered many times on this site, so it is certainly a duplicate. On the other hand, it may be still worth to give the link to a very nice lecture note by Keith Conrad here, where he does proofs for many interesting cases of the Mordell equation $y^2=x^3-n$. For $n=5$ there are no integer solutions. More references here are found at OEIS A054504.






      share|cite|improve this answer









      $endgroup$













      • $begingroup$
        In this post such ideals are considered in Arturo's answer, but there are still better posts, I suppose.
        $endgroup$
        – Dietrich Burde
        Jan 25 at 16:37












      • $begingroup$
        You were almost there.
        $endgroup$
        – nguyen quang do
        Jan 27 at 10:28














      3












      3








      3





      $begingroup$

      This has been answered many times on this site, so it is certainly a duplicate. On the other hand, it may be still worth to give the link to a very nice lecture note by Keith Conrad here, where he does proofs for many interesting cases of the Mordell equation $y^2=x^3-n$. For $n=5$ there are no integer solutions. More references here are found at OEIS A054504.






      share|cite|improve this answer









      $endgroup$



      This has been answered many times on this site, so it is certainly a duplicate. On the other hand, it may be still worth to give the link to a very nice lecture note by Keith Conrad here, where he does proofs for many interesting cases of the Mordell equation $y^2=x^3-n$. For $n=5$ there are no integer solutions. More references here are found at OEIS A054504.







      share|cite|improve this answer












      share|cite|improve this answer



      share|cite|improve this answer










      answered Jan 25 at 16:22









      Dietrich BurdeDietrich Burde

      80.4k647104




      80.4k647104












      • $begingroup$
        In this post such ideals are considered in Arturo's answer, but there are still better posts, I suppose.
        $endgroup$
        – Dietrich Burde
        Jan 25 at 16:37












      • $begingroup$
        You were almost there.
        $endgroup$
        – nguyen quang do
        Jan 27 at 10:28


















      • $begingroup$
        In this post such ideals are considered in Arturo's answer, but there are still better posts, I suppose.
        $endgroup$
        – Dietrich Burde
        Jan 25 at 16:37












      • $begingroup$
        You were almost there.
        $endgroup$
        – nguyen quang do
        Jan 27 at 10:28
















      $begingroup$
      In this post such ideals are considered in Arturo's answer, but there are still better posts, I suppose.
      $endgroup$
      – Dietrich Burde
      Jan 25 at 16:37






      $begingroup$
      In this post such ideals are considered in Arturo's answer, but there are still better posts, I suppose.
      $endgroup$
      – Dietrich Burde
      Jan 25 at 16:37














      $begingroup$
      You were almost there.
      $endgroup$
      – nguyen quang do
      Jan 27 at 10:28




      $begingroup$
      You were almost there.
      $endgroup$
      – nguyen quang do
      Jan 27 at 10:28











      1












      $begingroup$

      In the ideal factorization $(x+sqrt {-5})(x-sqrt {-5})=(y)^3$, "how can I show that the ideals $(x pm sqrt {-5})$ are relatively prime?" Suppose that $P$ is a common prime divisor, then $P^{2a}$ will appear in the LHS of the factorization. But $P^{3b}$ will appear in the RHS because of the cube, hence at least $P^6$ will divide both sides because of uniqueness. Following your argument, $P^3$ must divide $(2)(sqrt {-5})$. In the quadratic field $mathbf Q(sqrt {-5})$, we know that the prime $5$ is totally ramified, and $(5)=(sqrt {-5})^2$ shows that $(sqrt {-5})$ is prime. Besides $(2)=Q^2$, with $Q=(2, 1+sqrt {-5})$ (so the class of $Q$ is the unique class of order $2$ of the class group). It follows again by uniqueness of factorization that $P^3$ cannot divide $(sqrt {-5})Q^2$, and we are done.






      share|cite|improve this answer









      $endgroup$


















        1












        $begingroup$

        In the ideal factorization $(x+sqrt {-5})(x-sqrt {-5})=(y)^3$, "how can I show that the ideals $(x pm sqrt {-5})$ are relatively prime?" Suppose that $P$ is a common prime divisor, then $P^{2a}$ will appear in the LHS of the factorization. But $P^{3b}$ will appear in the RHS because of the cube, hence at least $P^6$ will divide both sides because of uniqueness. Following your argument, $P^3$ must divide $(2)(sqrt {-5})$. In the quadratic field $mathbf Q(sqrt {-5})$, we know that the prime $5$ is totally ramified, and $(5)=(sqrt {-5})^2$ shows that $(sqrt {-5})$ is prime. Besides $(2)=Q^2$, with $Q=(2, 1+sqrt {-5})$ (so the class of $Q$ is the unique class of order $2$ of the class group). It follows again by uniqueness of factorization that $P^3$ cannot divide $(sqrt {-5})Q^2$, and we are done.






        share|cite|improve this answer









        $endgroup$
















          1












          1








          1





          $begingroup$

          In the ideal factorization $(x+sqrt {-5})(x-sqrt {-5})=(y)^3$, "how can I show that the ideals $(x pm sqrt {-5})$ are relatively prime?" Suppose that $P$ is a common prime divisor, then $P^{2a}$ will appear in the LHS of the factorization. But $P^{3b}$ will appear in the RHS because of the cube, hence at least $P^6$ will divide both sides because of uniqueness. Following your argument, $P^3$ must divide $(2)(sqrt {-5})$. In the quadratic field $mathbf Q(sqrt {-5})$, we know that the prime $5$ is totally ramified, and $(5)=(sqrt {-5})^2$ shows that $(sqrt {-5})$ is prime. Besides $(2)=Q^2$, with $Q=(2, 1+sqrt {-5})$ (so the class of $Q$ is the unique class of order $2$ of the class group). It follows again by uniqueness of factorization that $P^3$ cannot divide $(sqrt {-5})Q^2$, and we are done.






          share|cite|improve this answer









          $endgroup$



          In the ideal factorization $(x+sqrt {-5})(x-sqrt {-5})=(y)^3$, "how can I show that the ideals $(x pm sqrt {-5})$ are relatively prime?" Suppose that $P$ is a common prime divisor, then $P^{2a}$ will appear in the LHS of the factorization. But $P^{3b}$ will appear in the RHS because of the cube, hence at least $P^6$ will divide both sides because of uniqueness. Following your argument, $P^3$ must divide $(2)(sqrt {-5})$. In the quadratic field $mathbf Q(sqrt {-5})$, we know that the prime $5$ is totally ramified, and $(5)=(sqrt {-5})^2$ shows that $(sqrt {-5})$ is prime. Besides $(2)=Q^2$, with $Q=(2, 1+sqrt {-5})$ (so the class of $Q$ is the unique class of order $2$ of the class group). It follows again by uniqueness of factorization that $P^3$ cannot divide $(sqrt {-5})Q^2$, and we are done.







          share|cite|improve this answer












          share|cite|improve this answer



          share|cite|improve this answer










          answered Jan 27 at 18:16









          nguyen quang donguyen quang do

          8,9891724




          8,9891724























              1












              $begingroup$

              If ideals $I,J$ are relatively prime then their sum generates the ring $R$, so we can show this. In particular we can show also that
              $$
              (a + bsqrt {- 5})(x+ sqrt{- 5}) + (c + dsqrt {- 5})(x - sqrt {- 5}) = 1
              $$

              for some $a,b,c,d$.



              This gives the equations
              $$
              begin{align}
              (a+c)x - 5(b-d) &= 1\
              (a-c) + (b+d)x &= 0
              end{align}
              $$

              So a necessary condition is $5$ does not divide $x$. This can be checked to be true from $x^2 + 5 = y^3$, but otherwise the ideals are not necessarily relatively prime in general.





              Setting
              $$
              c = a + (b+d)x
              $$

              gives us
              $$
              2ax = (1+5(b-d)) - (b+d)x^2
              $$

              Taking $pmod 2$ we see that $x$ must be even, so to get an integer solution for $a$ we must have $1+5(b-d)$ divisible by $2x$.



              The obvious way is to look at the linear diophantine solutions
              $$
              xu - 5v = 1
              $$

              We can always choose $u$ to be even, so now we can set $d=0, v=b$ to get
              $$
              begin{align}
              a &= frac{1+5(b-d)}{2x} - (b+d)frac{x}{2}\
              &= frac{1+5v}{2x} - vfrac{x}{2}\
              &= frac{u}{2} - vfrac{x}{2}
              end{align}
              $$

              Since $u/2$ is integral we just need $vx$ to be even, which is true based on an earlier assumption: $x$ must be even.





              It remains to show that $x$ is indeed even.
              If $x=2m+1$ is odd, then $y$ is even and
              $$
              x^2 + 5 = 4m^2+4m+ 6 equiv 2pmod 4
              $$

              which contradicts $y^3 equiv 0 pmod 4$. Hence $x$ is even and we are done, finding a solution to the initial equation as
              $$
              (u/2 -vx/2 + vsqrt {-5})(x + sqrt {-5}) + (u/2 + vx/2)(x - sqrt {-5}) = 1
              $$

              (Recall that $xu-5v=1$ and $u$ chosen to be even.)



              Since the two ideals generate $1$, they are relatively prime.






              share|cite|improve this answer











              $endgroup$


















                1












                $begingroup$

                If ideals $I,J$ are relatively prime then their sum generates the ring $R$, so we can show this. In particular we can show also that
                $$
                (a + bsqrt {- 5})(x+ sqrt{- 5}) + (c + dsqrt {- 5})(x - sqrt {- 5}) = 1
                $$

                for some $a,b,c,d$.



                This gives the equations
                $$
                begin{align}
                (a+c)x - 5(b-d) &= 1\
                (a-c) + (b+d)x &= 0
                end{align}
                $$

                So a necessary condition is $5$ does not divide $x$. This can be checked to be true from $x^2 + 5 = y^3$, but otherwise the ideals are not necessarily relatively prime in general.





                Setting
                $$
                c = a + (b+d)x
                $$

                gives us
                $$
                2ax = (1+5(b-d)) - (b+d)x^2
                $$

                Taking $pmod 2$ we see that $x$ must be even, so to get an integer solution for $a$ we must have $1+5(b-d)$ divisible by $2x$.



                The obvious way is to look at the linear diophantine solutions
                $$
                xu - 5v = 1
                $$

                We can always choose $u$ to be even, so now we can set $d=0, v=b$ to get
                $$
                begin{align}
                a &= frac{1+5(b-d)}{2x} - (b+d)frac{x}{2}\
                &= frac{1+5v}{2x} - vfrac{x}{2}\
                &= frac{u}{2} - vfrac{x}{2}
                end{align}
                $$

                Since $u/2$ is integral we just need $vx$ to be even, which is true based on an earlier assumption: $x$ must be even.





                It remains to show that $x$ is indeed even.
                If $x=2m+1$ is odd, then $y$ is even and
                $$
                x^2 + 5 = 4m^2+4m+ 6 equiv 2pmod 4
                $$

                which contradicts $y^3 equiv 0 pmod 4$. Hence $x$ is even and we are done, finding a solution to the initial equation as
                $$
                (u/2 -vx/2 + vsqrt {-5})(x + sqrt {-5}) + (u/2 + vx/2)(x - sqrt {-5}) = 1
                $$

                (Recall that $xu-5v=1$ and $u$ chosen to be even.)



                Since the two ideals generate $1$, they are relatively prime.






                share|cite|improve this answer











                $endgroup$
















                  1












                  1








                  1





                  $begingroup$

                  If ideals $I,J$ are relatively prime then their sum generates the ring $R$, so we can show this. In particular we can show also that
                  $$
                  (a + bsqrt {- 5})(x+ sqrt{- 5}) + (c + dsqrt {- 5})(x - sqrt {- 5}) = 1
                  $$

                  for some $a,b,c,d$.



                  This gives the equations
                  $$
                  begin{align}
                  (a+c)x - 5(b-d) &= 1\
                  (a-c) + (b+d)x &= 0
                  end{align}
                  $$

                  So a necessary condition is $5$ does not divide $x$. This can be checked to be true from $x^2 + 5 = y^3$, but otherwise the ideals are not necessarily relatively prime in general.





                  Setting
                  $$
                  c = a + (b+d)x
                  $$

                  gives us
                  $$
                  2ax = (1+5(b-d)) - (b+d)x^2
                  $$

                  Taking $pmod 2$ we see that $x$ must be even, so to get an integer solution for $a$ we must have $1+5(b-d)$ divisible by $2x$.



                  The obvious way is to look at the linear diophantine solutions
                  $$
                  xu - 5v = 1
                  $$

                  We can always choose $u$ to be even, so now we can set $d=0, v=b$ to get
                  $$
                  begin{align}
                  a &= frac{1+5(b-d)}{2x} - (b+d)frac{x}{2}\
                  &= frac{1+5v}{2x} - vfrac{x}{2}\
                  &= frac{u}{2} - vfrac{x}{2}
                  end{align}
                  $$

                  Since $u/2$ is integral we just need $vx$ to be even, which is true based on an earlier assumption: $x$ must be even.





                  It remains to show that $x$ is indeed even.
                  If $x=2m+1$ is odd, then $y$ is even and
                  $$
                  x^2 + 5 = 4m^2+4m+ 6 equiv 2pmod 4
                  $$

                  which contradicts $y^3 equiv 0 pmod 4$. Hence $x$ is even and we are done, finding a solution to the initial equation as
                  $$
                  (u/2 -vx/2 + vsqrt {-5})(x + sqrt {-5}) + (u/2 + vx/2)(x - sqrt {-5}) = 1
                  $$

                  (Recall that $xu-5v=1$ and $u$ chosen to be even.)



                  Since the two ideals generate $1$, they are relatively prime.






                  share|cite|improve this answer











                  $endgroup$



                  If ideals $I,J$ are relatively prime then their sum generates the ring $R$, so we can show this. In particular we can show also that
                  $$
                  (a + bsqrt {- 5})(x+ sqrt{- 5}) + (c + dsqrt {- 5})(x - sqrt {- 5}) = 1
                  $$

                  for some $a,b,c,d$.



                  This gives the equations
                  $$
                  begin{align}
                  (a+c)x - 5(b-d) &= 1\
                  (a-c) + (b+d)x &= 0
                  end{align}
                  $$

                  So a necessary condition is $5$ does not divide $x$. This can be checked to be true from $x^2 + 5 = y^3$, but otherwise the ideals are not necessarily relatively prime in general.





                  Setting
                  $$
                  c = a + (b+d)x
                  $$

                  gives us
                  $$
                  2ax = (1+5(b-d)) - (b+d)x^2
                  $$

                  Taking $pmod 2$ we see that $x$ must be even, so to get an integer solution for $a$ we must have $1+5(b-d)$ divisible by $2x$.



                  The obvious way is to look at the linear diophantine solutions
                  $$
                  xu - 5v = 1
                  $$

                  We can always choose $u$ to be even, so now we can set $d=0, v=b$ to get
                  $$
                  begin{align}
                  a &= frac{1+5(b-d)}{2x} - (b+d)frac{x}{2}\
                  &= frac{1+5v}{2x} - vfrac{x}{2}\
                  &= frac{u}{2} - vfrac{x}{2}
                  end{align}
                  $$

                  Since $u/2$ is integral we just need $vx$ to be even, which is true based on an earlier assumption: $x$ must be even.





                  It remains to show that $x$ is indeed even.
                  If $x=2m+1$ is odd, then $y$ is even and
                  $$
                  x^2 + 5 = 4m^2+4m+ 6 equiv 2pmod 4
                  $$

                  which contradicts $y^3 equiv 0 pmod 4$. Hence $x$ is even and we are done, finding a solution to the initial equation as
                  $$
                  (u/2 -vx/2 + vsqrt {-5})(x + sqrt {-5}) + (u/2 + vx/2)(x - sqrt {-5}) = 1
                  $$

                  (Recall that $xu-5v=1$ and $u$ chosen to be even.)



                  Since the two ideals generate $1$, they are relatively prime.







                  share|cite|improve this answer














                  share|cite|improve this answer



                  share|cite|improve this answer








                  edited Jan 28 at 2:18

























                  answered Jan 27 at 12:35









                  Yong Hao NgYong Hao Ng

                  3,5691222




                  3,5691222






























                      draft saved

                      draft discarded




















































                      Thanks for contributing an answer to Mathematics Stack Exchange!


                      • Please be sure to answer the question. Provide details and share your research!

                      But avoid



                      • Asking for help, clarification, or responding to other answers.

                      • Making statements based on opinion; back them up with references or personal experience.


                      Use MathJax to format equations. MathJax reference.


                      To learn more, see our tips on writing great answers.




                      draft saved


                      draft discarded














                      StackExchange.ready(
                      function () {
                      StackExchange.openid.initPostLogin('.new-post-login', 'https%3a%2f%2fmath.stackexchange.com%2fquestions%2f3087254%2fshowing-that-x25-y3-has-no-integer-solutions%23new-answer', 'question_page');
                      }
                      );

                      Post as a guest















                      Required, but never shown





















































                      Required, but never shown














                      Required, but never shown












                      Required, but never shown







                      Required, but never shown

































                      Required, but never shown














                      Required, but never shown












                      Required, but never shown







                      Required, but never shown







                      Popular posts from this blog

                      Mario Kart Wii

                      What does “Dominus providebit” mean?

                      Antonio Litta Visconti Arese